Difference between revisions of "1987 AJHSME Problems"

(Problem 3)
(Problem 3)
 
(30 intermediate revisions by 9 users not shown)
Line 1: Line 1:
 +
{{AJHSME Problems
 +
|year = 1987
 +
}}
 
== Problem 1 ==
 
== Problem 1 ==
  
Line 24: Line 27:
  
 
== Problem 4 ==
 
== Problem 4 ==
 +
 +
Martians measure angles in clerts.  There are <math>500</math> clerts in a full circle.  How many clerts are there in a right angle?
 +
 +
<math>\text{(A)}\ 90 \qquad \text{(B)}\ 100 \qquad \text{(C)}\ 125 \qquad \text{(D)}\ 180 \qquad \text{(E)}\ 250</math>
  
 
[[1987 AJHSME Problems/Problem 4|Solution]]
 
[[1987 AJHSME Problems/Problem 4|Solution]]
  
 
== Problem 5 ==
 
== Problem 5 ==
 +
 +
The area of the rectangular region is
 +
 +
<asy>
 +
draw((0,0)--(4,0)--(4,2.2)--(0,2.2)--cycle,linewidth(.5 mm));
 +
label(".22 m",(4,1.1),E);
 +
label(".4 m",(2,0),S);
 +
</asy>
 +
 +
<math>\text{(A)}\ \text{.088 m}^2 \qquad \text{(B)}\ \text{.62 m}^2 \qquad \text{(C)}\ \text{.88 m}^2 \qquad \text{(D)}\ \text{1.24 m}^2 \qquad \text{(E)}\ \text{4.22 m}^2</math>
  
 
[[1987 AJHSME Problems/Problem 5|Solution]]
 
[[1987 AJHSME Problems/Problem 5|Solution]]
  
 
== Problem 6 ==
 
== Problem 6 ==
 +
 +
The smallest product one could obtain by multiplying two numbers in the set <math>\{ -7, -5, -1, 1, 3 \}</math> is
 +
 +
<math>\text{(A)}\ -35 \qquad \text{(B)}\ -21 \qquad \text{(C)}\ -15 \qquad \text{(D)}\ -1 \qquad \text{(E)}\ 3</math>
  
 
[[1987 AJHSME Problems/Problem 6|Solution]]
 
[[1987 AJHSME Problems/Problem 6|Solution]]
  
 
== Problem 7 ==
 
== Problem 7 ==
 +
 +
The large cube shown is made up of <math>27</math> identical sized smaller cubes.  For each face of the large cube, the opposite face is shaded the same way.  The total number of smaller cubes that must have at least one face shaded is
 +
 +
<math>\text{(A)}\ 10 \qquad \text{(B)}\ 16 \qquad \text{(C)}\ 20 \qquad \text{(D)}\ 22 \qquad \text{(E)}\ 24</math>
 +
 +
<asy>
 +
unitsize(36);
 +
draw((0,0)--(3,0)--(3,3)--(0,3)--cycle);
 +
draw((3,0)--(5.2,1.4)--(5.2,4.4)--(3,3));
 +
draw((0,3)--(2.2,4.4)--(5.2,4.4));
 +
fill((0,0)--(0,1)--(1,1)--(1,0)--cycle,black);
 +
fill((0,2)--(0,3)--(1,3)--(1,2)--cycle,black);
 +
fill((1,1)--(1,2)--(2,2)--(2,1)--cycle,black);
 +
fill((2,0)--(3,0)--(3,1)--(2,1)--cycle,black);
 +
fill((2,2)--(3,2)--(3,3)--(2,3)--cycle,black);
 +
draw((1,3)--(3.2,4.4));
 +
draw((2,3)--(4.2,4.4));
 +
draw((.733333333,3.4666666666)--(3.73333333333,3.466666666666));
 +
draw((1.466666666,3.9333333333)--(4.466666666,3.9333333333));
 +
fill((1.73333333,3.46666666666)--(2.7333333333,3.46666666666)--(3.46666666666,3.93333333333)--(2.46666666666,3.93333333333)--cycle,black);
 +
fill((3,1)--(3.733333333333,1.466666666666)--(3.73333333333,2.46666666666)--(3,2)--cycle,black);
 +
fill((3.73333333333,.466666666666)--(4.466666666666,.93333333333)--(4.46666666666,1.93333333333)--(3.733333333333,1.46666666666)--cycle,black);
 +
fill((3.73333333333,2.466666666666)--(4.466666666666,2.93333333333)--(4.46666666666,3.93333333333)--(3.733333333333,3.46666666666)--cycle,black);
 +
fill((4.466666666666,1.9333333333333)--(5.2,2.4)--(5.2,3.4)--(4.4666666666666,2.9333333333333)--cycle,black);
 +
</asy>
  
 
[[1987 AJHSME Problems/Problem 7|Solution]]
 
[[1987 AJHSME Problems/Problem 7|Solution]]
  
 
== Problem 8 ==
 
== Problem 8 ==
 +
 +
If <math>\text{A}</math> and <math>\text{B}</math> are nonzero digits, then the number of digits (not necessarily different) in the sum of the three whole numbers is
 +
 +
<cmath>\begin{tabular}[t]{cccc}
 +
9 & 8 & 7 & 6 \\
 +
& A & 3 & 2 \\
 +
& & B & 1 \\ \hline
 +
\end{tabular}</cmath>
 +
 +
 +
<math>\text{(A)}\ 4 \qquad \text{(B)}\ 5 \qquad \text{(C)}\ 6 \qquad \text{(D)}\ 9 \qquad \text{(E)}\ \text{depends on the values of A and B}</math>
  
 
[[1987 AJHSME Problems/Problem 8|Solution]]
 
[[1987 AJHSME Problems/Problem 8|Solution]]
  
 
== Problem 9 ==
 
== Problem 9 ==
 +
 +
When finding the sum <math>\frac{1}{2}+\frac{1}{3}+\frac{1}{4}+\frac{1}{5}+\frac{1}{6}+\frac{1}{7}</math>, the least common denominator used is
 +
 +
<math>\text{(A)}\ 120 \qquad \text{(B)}\ 210 \qquad \text{(C)}\ 420 \qquad \text{(D)}\ 840 \qquad \text{(E)}\ 5040</math>
  
 
[[1987 AJHSME Problems/Problem 9|Solution]]
 
[[1987 AJHSME Problems/Problem 9|Solution]]
  
 
== Problem 10 ==
 
== Problem 10 ==
 +
 +
<math>4(299)+3(299)+2(299)+298=</math>
 +
 +
<math>\text{(A)}\ 2889 \qquad \text{(B)}\ 2989 \qquad \text{(C)}\ 2991 \qquad \text{(D)}\ 2999 \qquad \text{(E)}\ 3009</math>
  
 
[[1987 AJHSME Problems/Problem 10|Solution]]
 
[[1987 AJHSME Problems/Problem 10|Solution]]
  
 
== Problem 11 ==
 
== Problem 11 ==
 +
 +
The sum <math>2\frac17+3\frac12+5\frac{1}{19}</math> is between
 +
 +
<math>\text{(A)}\ 10\text{ and }10\frac12 \qquad \text{(B)}\ 10\frac12 \text{ and } 11 \qquad \text{(C)}\ 11\text{ and }11\frac12 \qquad \text{(D)}\ 11\frac12 \text{ and }12 \qquad \text{(E)}\ 12\text{ and }12\frac12</math>
  
 
[[1987 AJHSME Problems/Problem 11|Solution]]
 
[[1987 AJHSME Problems/Problem 11|Solution]]
  
 
== Problem 12 ==
 
== Problem 12 ==
 +
 +
What fraction of the large <math>12</math> by <math>18</math> rectangular region is shaded?
 +
 +
<asy>
 +
draw((0,0)--(18,0)--(18,12)--(0,12)--cycle);
 +
draw((0,6)--(18,6));
 +
for(int a=6; a<12; ++a)
 +
{
 +
  draw((1.5a,0)--(1.5a,6));
 +
}
 +
fill((15,0)--(18,0)--(18,6)--(15,6)--cycle,black);
 +
label("0",(0,0),W);
 +
label("9",(9,0),S);
 +
label("18",(18,0),S);
 +
label("6",(0,6),W);
 +
label("12",(0,12),W);
 +
</asy>
 +
 +
<math>\text{(A)}\ \frac{1}{108} \qquad \text{(B)}\ \frac{1}{18} \qquad \text{(C)}\ \frac{1}{12} \qquad \text{(D)}\ \frac29 \qquad \text{(E)}\ \frac13</math>
  
 
[[1987 AJHSME Problems/Problem 12|Solution]]
 
[[1987 AJHSME Problems/Problem 12|Solution]]
  
 
== Problem 13 ==
 
== Problem 13 ==
 +
 +
Which of the following fractions has the largest value?
 +
 +
<math>\text{(A)}\ \frac{3}{7} \qquad \text{(B)}\ \frac{4}{9} \qquad \text{(C)}\ \frac{17}{35} \qquad \text{(D)}\ \frac{100}{201} \qquad \text{(E)}\ \frac{151}{301}</math>
  
 
[[1987 AJHSME Problems/Problem 13|Solution]]
 
[[1987 AJHSME Problems/Problem 13|Solution]]
  
 
== Problem 14 ==
 
== Problem 14 ==
 +
 +
A computer can do <math>10,000</math> additions per second.  How many additions can it do in one hour?
 +
 +
<math>\text{(A)}\ 6\text{ million} \qquad \text{(B)}\ 36\text{ million} \qquad \text{(C)}\ 60\text{ million} \qquad \text{(D)}\ 216\text{ million} \qquad \text{(E)}\ 360\text{ million}</math>
  
 
[[1987 AJHSME Problems/Problem 14|Solution]]
 
[[1987 AJHSME Problems/Problem 14|Solution]]
  
 
== Problem 15 ==
 
== Problem 15 ==
 +
 +
The sale ad read: "Buy three tires at the regular price and get the fourth tire for three dollars;." Sam paid <math>240\text{ dollars}</math> for a set of four tires at the sale.  What was the regular price of one tire?
 +
 +
<math>\text{(A)}\ 59.25\text{ dollars} \qquad \text{(B)}\ 60\text{ dollars} \qquad \text{(C)}\ 70\text{ dollars} \qquad \text{(D)}\ 79\text{ dollars} \qquad \text{(E)}\ 80\text{ dollars}</math>
  
 
[[1987 AJHSME Problems/Problem 15|Solution]]
 
[[1987 AJHSME Problems/Problem 15|Solution]]
  
 
== Problem 16 ==
 
== Problem 16 ==
 +
 +
Joyce made <math>12</math> of her first <math>30</math> shots in the first three games of this basketball game, so her seasonal shooting average was <math>40\% </math>.  In her next game, she took <math>10</math> shots and raised her seasonal shooting average to <math>50\% </math>.  How many of these <math>10</math> shots did she make?
 +
 +
<math>\text{(A)}\ 2 \qquad \text{(B)}\ 3 \qquad \text{(C)}\ 5 \qquad \text{(D)}\ 6 \qquad \text{(E)}\ 8</math>
  
 
[[1987 AJHSME Problems/Problem 16|Solution]]
 
[[1987 AJHSME Problems/Problem 16|Solution]]
  
 
== Problem 17 ==
 
== Problem 17 ==
 +
 +
Abby, Bret, Carl, and Dana are seated in a row of four seats numbered #1 to #4.  Joe looks at them and says:
 +
 +
"Bret is next to Carl."
 +
"Abby is between Bret and Carl."
 +
 +
However each one of Joe's statements is false.  Bret is actually sitting in seat #3.  Who is sitting in seat #2?
 +
 +
<math>\text{(A)}\ \text{Abby} \qquad \text{(B)}\ \text{Bret} \qquad \text{(C)}\ \text{Carl} \qquad \text{(D)}\ \text{Dana} \qquad \text{(E)}\ \text{There is not enough information to be sure.}</math>
  
 
[[1987 AJHSME Problems/Problem 17|Solution]]
 
[[1987 AJHSME Problems/Problem 17|Solution]]
  
 
== Problem 18 ==
 
== Problem 18 ==
 +
 +
Half the people in a room left.  One third of those remaining started to dance.  There were then <math>12</math> people who were not dancing.  The original number of people in the room was what?
 +
 +
<math>\text{(A)}\ 24 \qquad \text{(B)}\ 30 \qquad \text{(C)}\ 36 \qquad \text{(D)}\ 42 \qquad \text{(E)}\ 72</math>
  
 
[[1987 AJHSME Problems/Problem 18|Solution]]
 
[[1987 AJHSME Problems/Problem 18|Solution]]
  
 
== Problem 19 ==
 
== Problem 19 ==
 +
 +
A calculator has a squaring key <math>\boxed{x^2}</math> which replaces the current number displayed with its square.  For example, if the display is <math>\boxed{000003}</math> and the <math>\boxed{x^2}</math> key is depressed, then the display becomes <math>\boxed{000009}</math>.  If the display reads <math>\boxed{000002}</math>, how many times must you depress the <math>\boxed{x^2}</math> key to produce a displayed number greater than <math>500</math>?
 +
 +
<math>\text{(A)}\ 4 \qquad \text{(B)}\ 5 \qquad \text{(C)}\ 8 \qquad \text{(D)}\ 9 \qquad \text{(E)}\ 250</math>
  
 
[[1987 AJHSME Problems/Problem 19|Solution]]
 
[[1987 AJHSME Problems/Problem 19|Solution]]
  
 
== Problem 20 ==
 
== Problem 20 ==
 +
 +
"If a whole number <math>n</math> is not prime, then the whole number <math>n-2</math> is not prime." A value of <math>n</math> which shows this statement to be false is
 +
 +
<math>\text{(A)}\ 9 \qquad \text{(B)}\ 12 \qquad \text{(C)}\ 13 \qquad \text{(D)}\ 16 \qquad \text{(E)}\ 23</math>
  
 
[[1987 AJHSME Problems/Problem 20|Solution]]
 
[[1987 AJHSME Problems/Problem 20|Solution]]
  
 
== Problem 21 ==
 
== Problem 21 ==
 +
 +
Suppose <math>n^{*}</math> means <math>\frac{1}{n}</math>, the reciprocal of <math>n</math>.  For example, <math>5^{*}=\frac{1}{5}</math>.  How many of the following statements are true?
 +
 +
i) <math>3^*+6^*=9^*</math>
 +
ii) <math>6^*-4^*=2^*</math>
 +
iii) <math>2^*\cdot 6^*=12^*</math>
 +
iv) <math>10^*\div 2^* =5^*</math>
 +
 +
<math>\text{(A)}\ 0 \qquad \text{(B)}\ 1 \qquad \text{(C)}\ 2 \qquad \text{(D)}\ 3 \qquad \text{(E)}\ 4</math>
  
 
[[1987 AJHSME Problems/Problem 21|Solution]]
 
[[1987 AJHSME Problems/Problem 21|Solution]]
  
 
== Problem 22 ==
 
== Problem 22 ==
 +
 +
<math>\text{ABCD}</math> is a rectangle, <math>\text{D}</math> is the center of the circle, and <math>\text{B}</math> is on the circle.  If <math>\text{AD}=4</math> and <math>\text{CD}=3</math>, then the area of the shaded region is between
 +
 +
<asy>
 +
pair A,B,C,D;
 +
A=(0,4); B=(3,4); C=(3,0); D=origin;
 +
draw(circle(D,5));
 +
fill((0,5)..(1.5,4.7697)..B--A--cycle,black);
 +
fill(B..(4,3)..(5,0)--C--cycle,black);
 +
draw((0,5)--D--(5,0));
 +
label("A",A,NW);
 +
label("B",B,NE);
 +
label("C",C,S);
 +
label("D",D,SW);
 +
</asy>
 +
 +
<math>\text{(A)}\ 4\text{ and }5 \qquad \text{(B)}\ 5\text{ and }6 \qquad \text{(C)}\ 6\text{ and }7 \qquad \text{(D)}\ 7\text{ and }8 \qquad \text{(E)}\ 8\text{ and }9</math>
  
 
[[1987 AJHSME Problems/Problem 22|Solution]]
 
[[1987 AJHSME Problems/Problem 22|Solution]]
  
 
== Problem 23 ==
 
== Problem 23 ==
 +
 +
Assume the adjoining chart shows the <math>1980</math> U.S. population, in millions, for each region by ethnic group.  To the nearest percent, what percent of the U.S. Black population lived in the South?
 +
 +
<cmath>\begin{tabular}[t]{c|cccc}
 +
& NE & MW & South & West \\ \hline
 +
White & 42 & 52 & 57 & 35 \\
 +
Black & 5 & 5 & 15 & 2 \\
 +
Asian & 1 & 1 & 1 & 3 \\
 +
Other & 1 & 1 & 2 & 4
 +
\end{tabular}</cmath>
 +
 +
<math>\text{(A)}\ 20\% \qquad \text{(B)}\ 25\% \qquad \text{(C)}\ 40\% \qquad \text{(D)}\ 56\% \qquad \text{(E)}\ 80\% </math>
  
 
[[1987 AJHSME Problems/Problem 23|Solution]]
 
[[1987 AJHSME Problems/Problem 23|Solution]]
  
 
== Problem 24 ==
 
== Problem 24 ==
 +
 +
A multiple choice examination consists of <math>20</math> questions.  The scoring is <math>+5</math> for each correct answer, <math>-2</math> for each incorrect answer, and <math>0</math> for each unanswered question.  John's score on the examination is <math>48</math>.  What is the maximum number of questions he could have answered correctly?
 +
 +
<math>\text{(A)}\ 9 \qquad \text{(B)}\ 10 \qquad \text{(C)}\ 11 \qquad \text{(D)}\ 12 \qquad \text{(E)}\ 16</math>
  
 
[[1987 AJHSME Problems/Problem 24|Solution]]
 
[[1987 AJHSME Problems/Problem 24|Solution]]
  
 
== Problem 25 ==
 
== Problem 25 ==
 +
 +
Ten balls numbered <math>1</math> to <math>10</math> are in a jar.  Jack reaches into the jar and randomly removes one of the balls.  Then Jill reaches into the jar and randomly removes a different ball.  The probability that the sum of the two numbers on the balls removed is even is
 +
 +
<math>\text{(A)}\ \frac{4}{9} \qquad \text{(B)}\ \frac{9}{19} \qquad \text{(C)}\ \frac{1}{2} \qquad \text{(D)}\ \frac{10}{19} \qquad \text{(E)}\ \frac{5}{9}</math>
  
 
[[1987 AJHSME Problems/Problem 25|Solution]]
 
[[1987 AJHSME Problems/Problem 25|Solution]]
  
== See also ==
+
== See Also ==
 +
{{AJHSME box|year=1987|before=[[1986 AJHSME Problems|1986 AJHSME]]|after=[[1988 AJHSME Problems|1988 AJHSME]]}}
 
* [[AJHSME]]
 
* [[AJHSME]]
 
* [[AJHSME Problems and Solutions]]
 
* [[AJHSME Problems and Solutions]]
* [[1987 AJHSME]]
 
 
* [[Mathematics competition resources]]
 
* [[Mathematics competition resources]]
 +
 +
 +
{{MAA Notice}}

Latest revision as of 09:35, 31 October 2021

1987 AJHSME (Answer Key)
Printable versions: WikiAoPS ResourcesPDF

Instructions

  1. This is a 25-question, multiple choice test. Each question is followed by answers marked A, B, C, D and E. Only one of these is correct.
  2. You will receive ? points for each correct answer, ? points for each problem left unanswered, and ? points for each incorrect answer.
  3. No aids are permitted other than scratch paper, graph paper, ruler, compass, protractor and erasers.
  4. Figures are not necessarily drawn to scale.
  5. You will have ? minutes working time to complete the test.
1 2 3 4 5 6 7 8 9 10 11 12 13 14 15 16 17 18 19 20 21 22 23 24 25

Problem 1

$.4+.02+.006=$

$\text{(A)}\ .012 \qquad \text{(B)}\ .066 \qquad \text{(C)}\ .12 \qquad \text{(D)}\ .24 \qquad \text{(E)} .426$

Solution

Problem 2

$\frac{2}{25}=$

$\text{(A)}\ .008 \qquad \text{(B)}\ .08 \qquad \text{(C)}\ .8 \qquad \text{(D)} 1.25 \qquad \text{(E)}\ 12.5$

Solution

Problem 3

$2(81+83+85+87+89+91+93+95+97+99)=$

$\text{(A)}\ 1600 \qquad \text{(B)}\ 1650 \qquad \text{(C)}\ 1700 \qquad \text{(D)}\ 1750 \qquad \text{(E)}\ 1800$

Solution

Problem 4

Martians measure angles in clerts. There are $500$ clerts in a full circle. How many clerts are there in a right angle?

$\text{(A)}\ 90 \qquad \text{(B)}\ 100 \qquad \text{(C)}\ 125 \qquad \text{(D)}\ 180 \qquad \text{(E)}\ 250$

Solution

Problem 5

The area of the rectangular region is

[asy] draw((0,0)--(4,0)--(4,2.2)--(0,2.2)--cycle,linewidth(.5 mm)); label(".22 m",(4,1.1),E); label(".4 m",(2,0),S); [/asy]

$\text{(A)}\ \text{.088 m}^2 \qquad \text{(B)}\ \text{.62 m}^2 \qquad \text{(C)}\ \text{.88 m}^2 \qquad \text{(D)}\ \text{1.24 m}^2 \qquad \text{(E)}\ \text{4.22 m}^2$

Solution

Problem 6

The smallest product one could obtain by multiplying two numbers in the set $\{ -7, -5, -1, 1, 3 \}$ is

$\text{(A)}\ -35 \qquad \text{(B)}\ -21 \qquad \text{(C)}\ -15 \qquad \text{(D)}\ -1 \qquad \text{(E)}\ 3$

Solution

Problem 7

The large cube shown is made up of $27$ identical sized smaller cubes. For each face of the large cube, the opposite face is shaded the same way. The total number of smaller cubes that must have at least one face shaded is

$\text{(A)}\ 10 \qquad \text{(B)}\ 16 \qquad \text{(C)}\ 20 \qquad \text{(D)}\ 22 \qquad \text{(E)}\ 24$

[asy] unitsize(36); draw((0,0)--(3,0)--(3,3)--(0,3)--cycle); draw((3,0)--(5.2,1.4)--(5.2,4.4)--(3,3)); draw((0,3)--(2.2,4.4)--(5.2,4.4)); fill((0,0)--(0,1)--(1,1)--(1,0)--cycle,black); fill((0,2)--(0,3)--(1,3)--(1,2)--cycle,black); fill((1,1)--(1,2)--(2,2)--(2,1)--cycle,black); fill((2,0)--(3,0)--(3,1)--(2,1)--cycle,black); fill((2,2)--(3,2)--(3,3)--(2,3)--cycle,black); draw((1,3)--(3.2,4.4)); draw((2,3)--(4.2,4.4)); draw((.733333333,3.4666666666)--(3.73333333333,3.466666666666)); draw((1.466666666,3.9333333333)--(4.466666666,3.9333333333)); fill((1.73333333,3.46666666666)--(2.7333333333,3.46666666666)--(3.46666666666,3.93333333333)--(2.46666666666,3.93333333333)--cycle,black); fill((3,1)--(3.733333333333,1.466666666666)--(3.73333333333,2.46666666666)--(3,2)--cycle,black); fill((3.73333333333,.466666666666)--(4.466666666666,.93333333333)--(4.46666666666,1.93333333333)--(3.733333333333,1.46666666666)--cycle,black); fill((3.73333333333,2.466666666666)--(4.466666666666,2.93333333333)--(4.46666666666,3.93333333333)--(3.733333333333,3.46666666666)--cycle,black); fill((4.466666666666,1.9333333333333)--(5.2,2.4)--(5.2,3.4)--(4.4666666666666,2.9333333333333)--cycle,black); [/asy]

Solution

Problem 8

If $\text{A}$ and $\text{B}$ are nonzero digits, then the number of digits (not necessarily different) in the sum of the three whole numbers is

\[\begin{tabular}[t]{cccc} 9 & 8 & 7 & 6 \\ & A & 3 & 2 \\ & & B & 1 \\ \hline  \end{tabular}\]


$\text{(A)}\ 4 \qquad \text{(B)}\ 5 \qquad \text{(C)}\ 6 \qquad \text{(D)}\ 9 \qquad \text{(E)}\ \text{depends on the values of A and B}$

Solution

Problem 9

When finding the sum $\frac{1}{2}+\frac{1}{3}+\frac{1}{4}+\frac{1}{5}+\frac{1}{6}+\frac{1}{7}$, the least common denominator used is

$\text{(A)}\ 120 \qquad \text{(B)}\ 210 \qquad \text{(C)}\ 420 \qquad \text{(D)}\ 840 \qquad \text{(E)}\ 5040$

Solution

Problem 10

$4(299)+3(299)+2(299)+298=$

$\text{(A)}\ 2889 \qquad \text{(B)}\ 2989 \qquad \text{(C)}\ 2991 \qquad \text{(D)}\ 2999 \qquad \text{(E)}\ 3009$

Solution

Problem 11

The sum $2\frac17+3\frac12+5\frac{1}{19}$ is between

$\text{(A)}\ 10\text{ and }10\frac12 \qquad \text{(B)}\ 10\frac12 \text{ and } 11 \qquad \text{(C)}\ 11\text{ and }11\frac12 \qquad \text{(D)}\ 11\frac12 \text{ and }12 \qquad \text{(E)}\ 12\text{ and }12\frac12$

Solution

Problem 12

What fraction of the large $12$ by $18$ rectangular region is shaded?

[asy] draw((0,0)--(18,0)--(18,12)--(0,12)--cycle); draw((0,6)--(18,6)); for(int a=6; a<12; ++a)  {   draw((1.5a,0)--(1.5a,6));  } fill((15,0)--(18,0)--(18,6)--(15,6)--cycle,black); label("0",(0,0),W); label("9",(9,0),S); label("18",(18,0),S); label("6",(0,6),W); label("12",(0,12),W); [/asy]

$\text{(A)}\ \frac{1}{108} \qquad \text{(B)}\ \frac{1}{18} \qquad \text{(C)}\ \frac{1}{12} \qquad \text{(D)}\ \frac29 \qquad \text{(E)}\ \frac13$

Solution

Problem 13

Which of the following fractions has the largest value?

$\text{(A)}\ \frac{3}{7} \qquad \text{(B)}\ \frac{4}{9} \qquad \text{(C)}\ \frac{17}{35} \qquad \text{(D)}\ \frac{100}{201} \qquad \text{(E)}\ \frac{151}{301}$

Solution

Problem 14

A computer can do $10,000$ additions per second. How many additions can it do in one hour?

$\text{(A)}\ 6\text{ million} \qquad \text{(B)}\ 36\text{ million} \qquad \text{(C)}\ 60\text{ million} \qquad \text{(D)}\ 216\text{ million} \qquad \text{(E)}\ 360\text{ million}$

Solution

Problem 15

The sale ad read: "Buy three tires at the regular price and get the fourth tire for three dollars;." Sam paid $240\text{ dollars}$ for a set of four tires at the sale. What was the regular price of one tire?

$\text{(A)}\ 59.25\text{ dollars} \qquad \text{(B)}\ 60\text{ dollars} \qquad \text{(C)}\ 70\text{ dollars} \qquad \text{(D)}\ 79\text{ dollars} \qquad \text{(E)}\ 80\text{ dollars}$

Solution

Problem 16

Joyce made $12$ of her first $30$ shots in the first three games of this basketball game, so her seasonal shooting average was $40\%$. In her next game, she took $10$ shots and raised her seasonal shooting average to $50\%$. How many of these $10$ shots did she make?

$\text{(A)}\ 2 \qquad \text{(B)}\ 3 \qquad \text{(C)}\ 5 \qquad \text{(D)}\ 6 \qquad \text{(E)}\ 8$

Solution

Problem 17

Abby, Bret, Carl, and Dana are seated in a row of four seats numbered #1 to #4. Joe looks at them and says:

"Bret is next to Carl."
"Abby is between Bret and Carl."

However each one of Joe's statements is false. Bret is actually sitting in seat #3. Who is sitting in seat #2?

$\text{(A)}\ \text{Abby} \qquad \text{(B)}\ \text{Bret} \qquad \text{(C)}\ \text{Carl} \qquad \text{(D)}\ \text{Dana} \qquad \text{(E)}\ \text{There is not enough information to be sure.}$

Solution

Problem 18

Half the people in a room left. One third of those remaining started to dance. There were then $12$ people who were not dancing. The original number of people in the room was what?

$\text{(A)}\ 24 \qquad \text{(B)}\ 30 \qquad \text{(C)}\ 36 \qquad \text{(D)}\ 42 \qquad \text{(E)}\ 72$

Solution

Problem 19

A calculator has a squaring key $\boxed{x^2}$ which replaces the current number displayed with its square. For example, if the display is $\boxed{000003}$ and the $\boxed{x^2}$ key is depressed, then the display becomes $\boxed{000009}$. If the display reads $\boxed{000002}$, how many times must you depress the $\boxed{x^2}$ key to produce a displayed number greater than $500$?

$\text{(A)}\ 4 \qquad \text{(B)}\ 5 \qquad \text{(C)}\ 8 \qquad \text{(D)}\ 9 \qquad \text{(E)}\ 250$

Solution

Problem 20

"If a whole number $n$ is not prime, then the whole number $n-2$ is not prime." A value of $n$ which shows this statement to be false is

$\text{(A)}\ 9 \qquad \text{(B)}\ 12 \qquad \text{(C)}\ 13 \qquad \text{(D)}\ 16 \qquad \text{(E)}\ 23$

Solution

Problem 21

Suppose $n^{*}$ means $\frac{1}{n}$, the reciprocal of $n$. For example, $5^{*}=\frac{1}{5}$. How many of the following statements are true?

i) $3^*+6^*=9^*$
ii) $6^*-4^*=2^*$
iii) $2^*\cdot 6^*=12^*$
iv) $10^*\div 2^* =5^*$

$\text{(A)}\ 0 \qquad \text{(B)}\ 1 \qquad \text{(C)}\ 2 \qquad \text{(D)}\ 3 \qquad \text{(E)}\ 4$

Solution

Problem 22

$\text{ABCD}$ is a rectangle, $\text{D}$ is the center of the circle, and $\text{B}$ is on the circle. If $\text{AD}=4$ and $\text{CD}=3$, then the area of the shaded region is between

[asy] pair A,B,C,D; A=(0,4); B=(3,4); C=(3,0); D=origin; draw(circle(D,5)); fill((0,5)..(1.5,4.7697)..B--A--cycle,black); fill(B..(4,3)..(5,0)--C--cycle,black); draw((0,5)--D--(5,0)); label("A",A,NW); label("B",B,NE); label("C",C,S); label("D",D,SW); [/asy]

$\text{(A)}\ 4\text{ and }5 \qquad \text{(B)}\ 5\text{ and }6 \qquad \text{(C)}\ 6\text{ and }7 \qquad \text{(D)}\ 7\text{ and }8 \qquad \text{(E)}\ 8\text{ and }9$

Solution

Problem 23

Assume the adjoining chart shows the $1980$ U.S. population, in millions, for each region by ethnic group. To the nearest percent, what percent of the U.S. Black population lived in the South?

\[\begin{tabular}[t]{c|cccc}  & NE & MW & South & West \\ \hline White & 42 & 52 & 57 & 35 \\ Black & 5 & 5 & 15 & 2 \\ Asian & 1 & 1 & 1 & 3 \\ Other & 1 & 1 & 2 & 4  \end{tabular}\]

$\text{(A)}\ 20\% \qquad \text{(B)}\ 25\% \qquad \text{(C)}\ 40\% \qquad \text{(D)}\ 56\% \qquad \text{(E)}\ 80\%$

Solution

Problem 24

A multiple choice examination consists of $20$ questions. The scoring is $+5$ for each correct answer, $-2$ for each incorrect answer, and $0$ for each unanswered question. John's score on the examination is $48$. What is the maximum number of questions he could have answered correctly?

$\text{(A)}\ 9 \qquad \text{(B)}\ 10 \qquad \text{(C)}\ 11 \qquad \text{(D)}\ 12 \qquad \text{(E)}\ 16$

Solution

Problem 25

Ten balls numbered $1$ to $10$ are in a jar. Jack reaches into the jar and randomly removes one of the balls. Then Jill reaches into the jar and randomly removes a different ball. The probability that the sum of the two numbers on the balls removed is even is

$\text{(A)}\ \frac{4}{9} \qquad \text{(B)}\ \frac{9}{19} \qquad \text{(C)}\ \frac{1}{2} \qquad \text{(D)}\ \frac{10}{19} \qquad \text{(E)}\ \frac{5}{9}$

Solution

See Also

1987 AJHSME (ProblemsAnswer KeyResources)
Preceded by
1986 AJHSME
Followed by
1988 AJHSME
1 2 3 4 5 6 7 8 9 10 11 12 13 14 15 16 17 18 19 20 21 22 23 24 25
All AJHSME/AMC 8 Problems and Solutions


The problems on this page are copyrighted by the Mathematical Association of America's American Mathematics Competitions. AMC logo.png